+1 Daumen
790 Aufrufe

Aufgabe:

Zeigen Sie mit Hilfe des Binomischen Satz, dass die folgenden Werte ganzzahlig sind:

a) \( ( 2 - \sqrt { 3 } ) ^ { n } + ( 2 + \sqrt { 3 } ) ^ { n } \) für jedes n ∈ ℕ

b) \( \sqrt { 2 } \left( ( \sqrt { 2 } + \sqrt { 5 } ) ^ { n } + ( \sqrt { 2 } - \sqrt { 5 } ) ^ { n } \right) \) für jedes ungerade n ∈ ℕ


Ich habe es bisher so gemacht:

$$ = \sum _ { k = 0 } ^ { n } \left( \begin{array} { c } { n } \\ { k } \end{array} \right) \cdot 2 ^ { k } \cdot ( - \sqrt { 3 } ) ^ { n - k } + \sum _ { k = 0 } ^ { n } \left( \begin{array} { c } { n } \\ { k } \end{array} \right) \cdot 2 ^ { k } \cdot ( \sqrt { 3 } ) ^ { n - k } = \sum _ { k = 0 } ^ { n } \frac { n ! \cdot 2 ^ { k } \cdot ( - \sqrt { 3 } ) ^ { n - k } } { ( n - k ) ! \cdot k ! } + \sum _ { k = 0 } ^ { n } \frac { n ! 2 ^ { k } \cdot ( \sqrt { 3 } ) ^ { n - k } } { ( n - k ) ! \cdot k ! } = \sum _ { k = 0 } ^ { n } \left( \frac { n ! \cdot 2 ^ { k } \cdot ( - \sqrt { 3 } ) ^ { n - k } + n ! \cdot 2 ^ { k } \cdot ( \sqrt { 3 } ) ^ { n - k } } { ( n - k ) ! \cdot k ! } \right) $$

Avatar von

2 Antworten

+1 Daumen

Das ist soweit richtig.

Den Binomialkoeffizienten ziehst du erstmal nochmal vor und diskutierst dann die Summanden abhängig von k:

$$ \sum _ { k = 0 } ^ { n } \left( \begin{array} { l } { n } \\ { k } \end{array} \right) 2 ^ { k } \left( ( - \sqrt { 3 } ) ^ { n - k } + ( \sqrt { 3 } ) ^ { n - k } \right) $$

Nun kannst du folgendermaßen argumentieren:

Wenn n-k ungerade ist, dann kannst du das Minuszeichen aus der linken Potenz rausziehen:
n-k = ungerade: (-√3)n-k + (√3)n-k = -(√3)n-k + (√3)n-k = 0

Das heißt diese Summanden fallen alle weg. Wenn aber n-k gerade ist, dann existiert eine ganze Zahl z mit der Eigenschaft 2z = n-k, also gilt:
(-√3)n-k + (√3)n-k = ((-√3)2)z + ((√3)2)z = 3z+3z = 2*3z

was eine ganze Zahl ist. Der Binomialkoeffizient ist auch eine ganze Zahl und 2k sowieso. Also ist jeder Summand Produkt von ganzen Zahlen und damit eine ganze Zahl. Die Summe ist dann Summe ganzer Zahlen und damit eine ganze Zahl.

b) ist ein bisschen subtiler, da muss ich nochmal drüber nachdenken :-)

---

"zu a) Wie geht es mit vollständiger Induktion. "

Ich denke nicht, beziehungsweise wenn, dann relativ kompliziert.

Hier ergibt sich der n+1-te Term ja nicht durch einfache Addition oder Multiplikation aus dem n-ten. Im Gegenteil, es müssen relativ viele Operationen durchgeführt werden, um einen Term in den nächsten zu überführen.

Der große Vorteil der vollständigen Induktion im Grunde nur den (einfachen) Schritt zu beweisen, ist also dahin.

Avatar von 10 k
0 Daumen

a) ist wurde ja schon von Julian Mi gelöst.

Nun zu b) mit n ungerade.

Aus √2 ^k mache ich da in den Zwischenschritten √2^n * √2^{k-n} = √2^n * (1/√(2^{n-k}).

$$ \sqrt { 2 } (\sum _{ k=0 }^{ n }{ \left( \begin{matrix} n \\ k \end{matrix} \right)  } \sqrt { 2 } { \quad  }^{ k }*\sqrt { 5 } { \quad  }^{ n-k }+\sum _{ k=0 }^{ n }{ \left( \begin{matrix} n \\ k \end{matrix} \right)  } \sqrt { 2 } { \quad  }^{ k }*(-\sqrt { 5 } ){ \quad  }^{ n-k })=\\ \sqrt { 2 } (\sum _{ k=0 }^{ n }{ \left( \begin{matrix} n \\ k \end{matrix} \right)  } { \sqrt { 2 }  }^{ n }(\sqrt { 2 } { \quad  }^{ k-n }*\sqrt { 5 } { \quad  }^{ n-k }+\sqrt { 2 } { \quad  }^{ k-n }*(-\sqrt { 5 } ){ \quad  }^{ n-k }))=\\ \sum _{ k=0 }^{ n }{ \left( \begin{matrix} n \\ k \end{matrix} \right)  } { \sqrt { 2 }  }^{ n+1 }({ \sqrt { \frac { 5 }{ 2 }  }  }^{ n-k }+\quad (-{ \sqrt { \frac { 5 }{ 2 }  } ) }^{ n-k }) $$

Hier ist nun n ungerade, also ist n+1 gerade und höchstens noch die Klammer irrational. ist aber n-k ungerade Subtrahieren sich die beiden Summanden weg. ist n-k gerade, addiert man 2 rationale Zahlen. Darum ist hier die Summe rational und dann bestimmt auch ganzzahlig, da 2^n/ 2^{n-k} wieder gekürzt werden kann. wegen k<n.

Avatar von 162 k 🚀
Wieso benutzt man bei der Aufgabe

a) k = 0

und bei b) k = 1?

Danke
k=0 ist wohl auch in b) schlauer. Allerdings kommt bei k=0 sicher nichts dazu, was nicht ganzzahlig ist. Hab das im Kommentar so geändert.

Ein anderes Problem?

Stell deine Frage

Willkommen bei der Mathelounge! Stell deine Frage einfach und kostenlos

x
Made by a lovely community